Massimi e minimi – Problema 7

Di tutti i rettangoli inscritti in uno stesso cerchio di raggio di misura R, qual’è quello di perimetro massimo? E di area massima?

Soluzione

Rappresentiamo un rettangolo qualunque inscritto in una circonferenza:

Vista la figura, chiamiamo \[ \overline{OB}=r\;;\;\overline{AB}=b\;;\;\overline{BC}=h \] \[ \overline{OH}=\frac{1}{2}\overline{AB}=\frac{b}{2} \] \[ \overline{BH}=\frac{1}{2}\overline{BC}=\frac{h}{2} \] Dal teorema di Pitagora sul triangolo OBH, otteniamo l’altezza del rettangolo h in funzione della base b: \[ \frac{h}{2}=\sqrt{r^{2}-\left(\frac{b}{2}\right)^{2}} \] \[ h=2\sqrt{r^{2}-\frac{b^{2}}{4}} \] \[ h=\sqrt{4r^{2}-b^{2}} \] Sapendo che il perimetro del rettangolo è \[ P=2\left(b+h\right) \] \[ P\left(b\right)=2\left(b+\sqrt{4r^{2}-b^{2}}\right) \] Ora la nostra funzione P ha una sola variabile (b), calcoliamo la derivata: \[ \frac{dP}{db}=2\left(1+\frac{-2b}{2\sqrt{4r^{2}-b^{2}}}\right) \] \[ \frac{dP}{db}=2\left(1-\frac{b}{\sqrt{4r^{2}-b^{2}}}\right) \] \[ \frac{dP}{db}=2\cdot\frac{\sqrt{4r^{2}-b^{2}}-b}{\sqrt{4r^{2}-b^{2}}} \] Studiamone il segno: \[ \frac{dP}{db}\geq0\rightarrow\sqrt{4r^{2}-b^{2}}-b\geq0 \] perchè il denominatore è positivo. Risulta quindi \[ \sqrt{4r^{2}-b^{2}}\geq b \] \[ 4r^{2}-b^{2}\geq b^{2} \] \[ 2b^{2}\leq4r^{2}\rightarrow b\leq\sqrt{2}r \] Abbiamo trovato l’intervallo di b per il quale la derivata della funzione P è positiva. La funzione P(b) è crescente per b minore di r{*}rad2, decrescente per b maggiore di r{*}rad2, ha quindi un massimo per \[ b=\sqrt{2}r \] \[ h=\sqrt{4r^{2}-b^{2}}\rightarrow h=\sqrt{2}r \] Quindi \[ b=h \] e il rettangolo inscritto per il quale si massimizza il perimetro è il quadrato.

Sapendo che l’area del rettangolo è \[ A=bh \] \[ A\left(b\right)=b\sqrt{4r^{2}-b^{2}} \] Ora la nostra funzione A ha una sola variabile (b), calcoliamo la derivata: \[ \frac{dA}{db}=\sqrt{4r^{2}-b^{2}}+b\frac{-2b}{2\sqrt{4r^{2}-b^{2}}} \] \[ \frac{dA}{db}=\sqrt{4r^{2}-b^{2}}-\frac{b^{2}}{\sqrt{4r^{2}-b^{2}}} \] \[ \frac{dA}{db}=\frac{4r^{2}-2b^{2}}{\sqrt{4r^{2}-b^{2}}} \] \[ \frac{dA}{db}=2\cdot\frac{2r^{2}-b^{2}}{\sqrt{4r^{2}-b^{2}}} \] Studiamone il segno: \[ \frac{dA}{db}\geq0\rightarrow2r^{2}-b^{2}\geq0 \] perchè il denominatore è positivo. Risulta quindi \[ 2r^{2}\geq b^{2}\rightarrow b\leq\sqrt{2}r \] Abbiamo trovato l’intervallo di b per il quale la derivata della funzione A è positiva. La funzione A(b) è crescente per b minore di r{*}rad2, decrescente per b maggiore di r{*}rad2, ha quindi un massimo per \[ b=\sqrt{2}r \] \[ h=\sqrt{4r^{2}-b^{2}}\rightarrow h=\sqrt{2}r \] Quindi \[ b=h \] e il rettangolo inscritto per il quale si massimizza l’area è il quadrato.

5 thoughts on “Massimi e minimi – Problema 7

  1. Ma a parte quel banale errore che io considero di distrazione (che comunque un pò di tempo me lo ha fatto perdere perchè, invece di provare a proseguire la lettura e scoprire l’errore, mi sono concentrato sul capire perchè OH fosse 1/2 AB) per il resto poi imposta tutto sulla relazione di Pitagora tra base e altezza e non vedo errori!

  2. @Pavlik
    è concettualmente sbagliato, poco importa che il risultato finale non cambi… a meno che non si voglia stimolare la revisione critica di chi legge queste dimostrazioni

    1. L’ho letto e riletto, ho anche provato a faro ma di concettualmente sbagliato non ho trovato nulla, a meno della svista iniziale dove sono stati scambiati OH e BH tra loro. Però poi è stato svolto nel modo corretto!

    1. Infatti è sbagliato: OH è h/2 e BH è b/2, ma dato che si vuole dimostrare che b=h non cambia molto.

Lascia un commento

Il tuo indirizzo email non sarà pubblicato. I campi obbligatori sono contrassegnati *